Seminarvortrag: Magnetische Monopole

Werbung
Seminarvortrag: Magnetische Monopole
Wolfgang Sebastian Lautz
4. Mai 2012
1
Motivation
~
~ + 1 ∂B = 0
∇×E
c ∂t
~ =0
∇B
4π ν
j
∂µ F µν =
c
~
~ − 1 ∂ E = 4π ~j
∇×B
c ∂t
c
~ = 4πρ
∇E
∂µ Feµν = 0
(Maxwell-Gleichungen)
(MWG kovariant)
Die klassische Elektrodynamik weist eine hohe Symmetrie von elektrischem und
magnetischem Feld auf: ein elektrisches Wirbelfeld induziert ein magnetisches
Wirbelfeld und so fort. Warum sollte diese Symmetrie zwischen elektrischem
und magnetischem Feld hinsichtlich der Quellen aufgehoben sein?
Im Rahmen der Maxwell’schen Theorie lassen sich magnetische Ladungen, d.h.
Monopole einführen. Klassisch spricht lediglich diese Symmetrieüberlegung für
die Einführung von magnetischen Monopolen. Vom Standpunkt der Quantenmechanik (bzw. in letzter Konsequenz der Quantenelektrodynamik) aus, hätte
die Existenz von magnetischen Monpolen eine weitere interessante Konsequenz:
die Quantisierung des Produktes von magnetischer sowie elektrischer Ladung.
Aus der Existenz einer kleinsten elektrischen Ladung folgt die Existenz einer
kleinsten magnetischen und umgekehrt.
2
Klassische Elektrodynamik mit magnetischen
Monopolen
Im folgenden soll die Möglichkeit untersucht werden, eine klassische dynamische
Theorie zu formulieren, welche punktförmige magnetische Monopole einschließt.
Hierzu verwenden wir den kovarianten Viererformalismus. Wir postulieren in
Analogie zur elektrischen Viererstromdichte einen magnetischen Viererstrom
k µ . Die Maxwellgleichungen nehmen dann die folgende Form an, wobei g die
magnetische Polstärke, d.h. die magnetische Ladung bezeichnet:
4π ν
j
cZ
dy ν 4
j ν (x) = e
δ (x − y (τ )) dτ
dτ
∂µ F µν =
4π ν
∂µ Feµν =
k
cZ
dz ν 4
k ν (x) = g
δ (x − z (τ )) dτ
dτ
1
(1)
(2)
2.1
Konstruktion eines 4er-Potentials
Nun muss allerdings die Definition des Viererpotentials modifiziert werden, denn
wenn man sich vorstellt, dass man einen ruhenden Monopol in den Ursprung
eines Koordinatensystems setzt, so folgt mithilfe des Gauß’schen Satzes:
Z
Z
~ = 4πgδ 4 (~x) ⇒
~ dA
~ = 4πg 6= 0 =
~ dV
∇B
B
∇ ∇×A
(3)
∂V
V
Dies gilt für beliebige Volumina V , welche den Monopol einschließen. Anderer~ = ∇ × A.
~ Dann kann A
~ nicht überall
seits gilt aber bis auf den Ursprung B
regulär sein, d.h. auf jeder den Monopol umschließenden Oberfläche muss eine
Singularität vorliegen, welche dazu beiträgt, dass das Integral (der Fluss) nicht
verschwindet. Es muss demnach eine sich vom Monopol ausgehende ins Unend~ singulär wird. Dies
liche erstreckende Linie geben, auf der das Vektorpotential A
ist der sogenannte Dirac-String.
Bewegt sich der Monopol längs irgendeiner Weltlinie im Minkowskiraum, so
muss dies zu allen Zeiten gelten, d.h. in vier Dimension erhält man eine sich halbseitig ins Unendliche erstreckende zweidimensionale Fläche (Raum und Zeit).
Dies ist das Sheet, welches wir nun wie folgt parametrisieren:
S : y = y (τ0 , τ1 ) ;
τ0 ∈ (−∞, ∞) ;
τ1 ∈ [0, ∞ )
(4)
Die Sheetparameter τ0 , τ1 werden hierbei so gewählt, dass für τ1 = 0 τ0 gerade
die Eigenzeit des Pols ist und somit gilt:
z (τ ) = y (τ0 , 0),
(5)
was gerade der Weltlinie des Pols entspricht. Es handelt sich hierbei um den
einzigen im Endlichen gelegenen Rand des Sheets.
Das Sheet ist lediglich ein Rechenhilfsmittel, keine physikalisch beobachtbare
Größe, worauf wir bei der Ableitung der Feldgleichungen1 noch zurückkommen
werden. Ansonsten ist die Wahl des Sheets frei. Unterschiedliche Strings führen
zu Potentialen, die sich lediglich um eine Eichtransformation unterscheiden.
Den Feldstärketensor definieren wir wie folgt:
F µν := ∂ µ Aν − ∂ ν Aµ +
4π e µν
G
c
4π µν
Feµν := εµναβ ∂α Aβ −
G
c
(6)
(7)
e µν überall mit Ausnahme des Sheets verschwindet. Dies setzen wir nun
wobei G
≈
in die zweite Maxwellgleichung ein unter Beachtung von G µν = − Gµν = Gνµ
(der Feldstärketensor ist antisymmetrisch):
∂µ
|
4π ν
∂µ Feµν =
k
c Z
4π
dz ν 4
4π µν
εµναβ ∂ α Aβ −
G )=
g
δ (x − z (τ )) dτ
c
dτ
{z
} c
(8)
(9)
=0
∂µ G
νµ
Z
=g
dz ν 4
δ (x − z (τ )) dτ
dτ
(10)
1 Dies wird uns auf das Dirac-Veto führen: Keine Ladung darf den String durchlaufen, da
sonst auch eine Wechselwirkung beobachtbar wäre.
2
Die Weltlinie des Pols ist der (endliche) Rand des Sheets:
z(τ ) = y (τ0 , 0) .
(11)
Hiermit kann man (9) als Integral über den Rand des Sheets schreiben und den
Satz von Stokes für die Ebene anwenden:
Z Z
∂v
∂v
∂u ∂v
∂u ∂v
u
dτ0 +
dτ1 =
−
dτ0 dτ1
(Stokes)
∂τ0
∂τ1
∂τ0 ∂τ1
∂τ1 ∂τ0
∂S
S
ν
∂y ν
∂y
dτ0 +
dτ1
δ 4 (x − y (τ0 , τ1 ))
∂τ0
∂τ1
∂S
Z ν
4
ν
4
∂y ∂δ (x − y) ∂y ∂δ (x − y)
= −g
−
dτ0 dτ1
∂τ0
∂τ1
∂τ1
∂τ0
S
Z ν
∂y ∂y µ
∂y ν ∂y µ ∂δ 4 (x − y)
= −g
dτ0 dτ1
−
∂τ0 ∂τ1
∂τ1 ∂τ0
∂y µ
S
Z ν
∂y ∂y µ
∂
∂y ν ∂y µ
4
=
g
−
δ
(x
−
y)
dτ
dτ
0
1
∂xµ
∂τ0 ∂τ1
∂τ1 ∂τ0
S
∂µ Gνµ = g
Z
(12)
(13)
(14)
(15)
Dabei wurde in den letzten beiden Schritten die Kettenregel angewandt und
statt nach y nach x differenziert. Insgesamt ergibt sich also:
Gνµ (x) = g
Z S
∂y ν ∂y µ
∂y ν ∂y µ
−
∂τ0 ∂τ1
∂τ1 ∂τ0
δ 4 (x − y) dτ0 dτ1
(16)
Der auf diese Weise modifizierte Feldstärketensor beschreibt nunmehr auch magnetische Monpole. Anschaulich haben wir erreicht, dass magnetische Feldlinien
nun einen nicht verschwindenden Nettofluss durch eine geschlossene Oberfläche
bewirken können, sie also nicht mehr zwangsläufig geschlossen sein müssen. Es
gibt Quellen und Senken.
2.2
Liénard-Wiechert Potential des Monopols
Eine bewegte elektrische (Punkt-)Ladung erzeugt ein elektromagnetisches Feld,
welche durch das Liénard-Wiechert Potential beschrieben wird:
Z
e
dzµ
Aµ (x) =
J (x − z (τ ))
dτ ,
(17)
c
dτ
wobei
J(x − z) = 4πδ 4 (x − z)
(18)
µ
J(x) = 2 Θ(x0 ) δ (xµ x )
(19)
die retardierte Greensfunktion darstellt. Dasselbe Verhalten erwarten wir von
bewegten magnetischen Monopolen. Hierbei nimmt das Liénard-Wiechert Potential die folgende Gestalt an:
g
Aν (x) = ενλρσ
c
Z
S
∂y λ ∂y ρ J (x − y (τ0 , τ1 ))
dτ0 dτ1
∂τ0 ∂τ1
∂xσ
3
(20)
Wir werden (20) nun beweisen, indem wir dies in (7) einsetzen. Es sollte ein
zum Liénard-Wiechert Potential der elektrischen Ladung (17) analoges Ergebnis liefern, in dem e und g miteinander vertauscht sind, denn wir ziehen unsere
gesamte Betrachtung nach dem Dualitätsprinzip auf: die MWG sowie die Gleichungen für die Quellen und damit auch die Potentiale ergeben sich durch die
Ersetzungen j ν → k ν sowie e → g.
Z
g µναβ
∂y λ ∂y ρ ∂ 2 J(x − y)
µναβ ∂Aν
=
ε
ε
dτ0 τ1
(21)
ε
νλρσ
∂xµ
c
∂y µ ∂yσ
S ∂τ0 ∂τ1
Z
g
∂y λ ∂y ρ ∂ 2 J(x − y) µ α β
=
δλ δρ δσ + δρµ δσα δλβ
c S ∂τ0 ∂τ1 ∂y µ ∂yσ
+δσµ δλα δρβ − (αβ) dτ0 dτ1
(22)
Dabei bedeutet (αβ) das die vorhergehenden Terme unter Vertauschung der
Indizes α und β subtrahiert werden.
Z µ
∂Aν
g
∂y ∂y α ∂ 2 J(x − y) ∂y β ∂y µ ∂ 2 J(x − y)
εµναβ
=
+
µ
∂x
c S ∂τ0 ∂τ1 ∂y µ ∂yβ
∂τ0 ∂τ1 ∂y µ ∂yα
∂y α ∂y β ∂ 2 J(x − y)
−(αβ) dτ0 dτ1
(23)
+
∂τ0 ∂τ1 ∂y µ ∂yµ
|
{z
}
=4πδ 4 (x−y)
∂y α ∂ 2 J(x − y) ∂y β ∂ 2 J(x − y)
+
∂τ0 ∂yβ
∂τ0 ∂τ1 ∂yα
S ∂τ1
4π αβ
−(αβ) dτ0 dτ1 +
G
c
Z
∂J(x − y) ∂y α
∂y α
Stokes g
=
dτ0 +
dτ1 − (αβ)
c ∂S
∂yβ
∂τ0
∂τ1
4π αβ
G
+
Zc
∂J(x − y) dz α
4π αβ
g
=
dτ − (αβ) +
G
c
∂yβ
dτ
c
=
g
c
Z (24)
(25)
(26)
Wobei wir im letzten Schritt wieder benutzt haben, dass y(τ0 , 0) = z(τ ) gerade
die Weltlinie des Pols ist und den einzigen nicht verschwindenden Beitrag zum
Integral über den Rand liefert. Insgesamt folgt, wenn man noch nach x statt
nach y differenziert:
Feαβ
4π αβ
= εµναβ ∂µ Aν −
G
c
Z
g
∂J(x − y) dz α
= −
dτ + (αβ)
c
∂xβ
dτ
=
∂ α B β (x) − ∂ β B α (x)
(27)
(28)
(29)
mit
B α (x) =
g
c
Z
J (x − z(τ ))
dz α
dτ
dτ
(30)
in völliger Analogie zum Potential einer bewegten elektrischen Ladung (17), wie
wir es auch erwartet hätten. Wir haben lediglich das Liénard-Wiechert Potential
4
in Abhängigkeit der Sheetparameter (20) ausgedrückt, da im nächsten Abschnitt
auch nach diesen variiert werden soll.
2.3
Lagrangeformalismus
In der klassischen Mechanik und Elektrodynamik ergeben sich die Bewegungsbzw. Feldgleichungen aus dem Hamilton’schen Prinzip, welches besagt, dass das
Wirkungsfunktional (im Falle einer Feldtheorie das Integral über die Lagrangedichte)
Z
S=
L (A, ∂µ A, x) d 4 x
(31)
V (4)
unter den üblichen Randbedingungen stationär wird:
δS = 0
wobei δx = 0,
δA = 0 |x∈∂V 4 .
(32)
Die Wirkung in der Elektrodynamik besteht aus drei Anteilen S = S1 + S2 + S3 ,
welche jeweils die (kinetische) Wirkung der Teilchen allein, die Wirkung des
elektromagnetischen Feldes 2 , sowie die Wechselwirkung der Ladungen mit dem
Feld (Wechselwirkung des Viererstroms) berücksichtigt. In der um magnetischen
Monopole erweiterten Theorie kann man das Wirkungsfunktional so beibehalten, es muss allerdings der modifizierte Feldstärktensor eingesetzt werden. Die
Variation der Wirkung soll nach den Koordinaten der Ladungen
bzw. Pole, den
P
Potentialen und den Sheetvariablen durchgeführt werden. e+g bedeutet hierbei, dass über alle Ladungen und Pole summiert wird.
r
XZ
dzµ dz µ
dτ
(33)
mc
S1 = −
dτ dτ
e+g
XZ
d 2 zµ µ
⇒ δS1 =
m
δz dτ
(34)
dτ 2
e+g
Z
1
S2 = −
Fµν (x)F µν (x) d 4 x
(35)
16π R4
Z
Z
1
g
dz ν µ
⇒ δS2 = −
∂ ν Fµν δAµ d 4 x −
Feµν (z)
δz dτ
4π R4
c ∂S
dτ
Z
∂y λ ∂y ν µ e
g
−
δy ∂λ Fµν + ∂µ Feνλ + ∂ν Feλµ dτ0 dτ1 (36)
c S ∂τ0 ∂τ1
XeZ
dzµ
S3 = −
Aµ (z)
dτ
(37)
c
dτ
e
i dz
XeZ h
µ
⇒ δS3 = −
(∂ ν Aµ − ∂ µ Aν ) δzν + δAµ
dτ
(38)
c
dτ
e
2 Dies schließt die Wechselwirkung der Pole mit dem Feld mit ein, wie wir sehen werden
und muss daher nicht unter S3 berücksichtigt werden
5
Insgesamt ergibt sich:
Z X
d 2 zµ
g
dz ν X e
δS =
m
− Feµν
−
(∂µ Aν − ∂νAµ )
2
dτ
c
dτ
c
e+g
e
Z X Z e dzµ
dz ν
1 ν
δz µ dτ +
∂ Fµν −
−
dτ
4π
c dτ
e
Z
∂y λ ∂y ν
g
δ 4 (x − z) dτ δAµ d 4 x −
c S ∂τ0 ∂τ1
∂λ Feµν + ∂µ Feνλ + ∂ν Feλµ δy µ dτ0 dτ1 = 0
(39)
Die Bewegungs- und Feldgleichungen erhält man, indem man die Koeffizienten
vor den unabhängigen Variationen gleich Null setzt:
d 2 zµ
dτ 2
d 2 zµ
Monopole: m
dτ 2
Ladungen: m
Feld: ∂ ν Fνµ
=
=
=
e
dz ν
(∂µ Aν − ∂ν Aµ )
c
dτ
g e dz ν
Fµν
c
Z dτ
4π
4π
dzµ 4
e
δ (x − z) dτ =
jµ
c
dτ
c
(40)
(41)
(42)
Aus der Variation der Sheetvariablen folgt:
∂λ Feµν + ∂µ Feνλ + ∂ν Feλµ
⇔
∂µ F
µν
=
⇔
=0
0
(auf dem Sheet)
ν
j ≡0
(43)
(44)
ν
Die zweite Maxwellgleichung ∂µ Feµν = 4π
c k gilt konstruktionsbedingt, aufgrund
unseres Ansatzes für den modifizierten Feldstärketensor, ähnlich wie in der Elektrodynamik ohne Pole, wo sich die quellfreie Maxwellgleichung allein schon aus
der Definition des Feldstärketensors als Viererrotation des Potentials ergibt. Aus
(44) folgt, das Dirac-Veto: eine Ladung darf zu keinem Zeitpunkt den String passieren, was sich auch in der Bewegungsgleichung (40) widerspiegelt, wenn man
e µν
an die Definition des Feldstärketensors denkt: F µν = ∂ µ Aν −∂ ν Aµ + 4π
c G . Der
G-Tensor verschwindet nämlich überall außer auf den Sheets. Die Bewegungsgleichungen für Pole und Ladungen sind somit strukturell gleich, man erhält
jeweils eine Lorentzkraft“, welche auf die Ladungsträger bzw. Pole wirkt.
”
Alles in allem ist es uns in diesem Abschnitt gelungen eine konsistente Elektrodynamik zu formulieren, welche magnetische Monpole zulässt.
3
Elektron im Feld eines ruhenden magnetischen
Monopols
Im Ursprung eines kartesischen Koordinatensystems ruhe ein magnetischer Monopol der Polstärke g. Wie wir bereits festgestellt haben (vgl. 2.1), kann es kein
~ für den ganzen R3 geben. Doch anvon Singularitäten freies Vektorpotential A
statt den Ansatz des Strings weiter zu verfolgen, wollen wir uns nun die lokale
Eichinvarianz zunutze machen. Wir zerlegen unseren Raum in zwei überlappende Gebiete G1 und G2 , welche zusammen den gesamten R3 überdecken. Auf
6
~ 1 bzw. A
~ 2 leben, welche sich
diesen Gebieten sollen jeweils Vektorpotentiale A
um den Gradienten eines Eichfeldes unterscheiden:
~1 = A
~ 2 − ∇χ
A
(45)
~ =
Für das B-Feld des Monopols gilt in Analogie zur Elektrostatik B
~
~
Kugelkoordinaten (r, ϑ, ϕ) schreibt sich die Rotation B = rotA als:
1
g
∂
∂Aϑ
=
(Aϕ sin ϑ) −
r2
r sin ϑ ∂ϑ
∂ϕ
1 ∂Ar
1 ∂
0 =
−
(rAϕ )
r sin ϑ ∂ϕ
r ∂r
∂Ar
1 ∂
(rAϑ ) −
0 =
r ∂r
∂ϑ
g
r2
~er . In
(46)
(47)
(48)
~ B,
~ d.h. Ar = 0.
Wir nutzen nun die Eichfreiheit des Potentials und fordern A⊥
1
Dann folgt aus (47) und (48), dass Aϕ , Aϑ ∝ r oder 0 sind. Wir wählen Aϑ = 0,
dies gibt eine besonders einfache Form des Potentials. Durch Integration der
radialen Komponente der Rotation (46), erhält man schließlich:
~ = g k − cos ϑ ~eϕ
(49)
A
r sin ϑ
k ist hierbei eine Integrationskonstante, deren Wahl einer bestimmten Eichung
~ für ϑ = 0, π singulär.
entspricht. In unserem Fall wählen wir k = ±1, dann ist A
Entsprechend wählen wir die Gebiete G1 : 0 ≤ ϑ < π2 + δ sowie G2 : π2 − δ <
ϑ ≤ π. Die dazugehörigen Vektorpotentiale sind
~ 1 = g 1 − cos ϑ ~eϕ
~ 2 = −g 1 + cos ϑ ~eϕ .
A
A
(50)
r sin ϑ
r sin ϑ
Das Eichfeld, um das sich die beiden Potentiale unterscheiden ist entsprechend:
2g
∇χ = −
~eϕ ⇒ χ = −2gϕ
(51)
r sin ϑ
Wir lassen nun ein Elektron in das Feld des Monopols eintreten und untersuchen
das Verhalten der Zustandskets und des Hamiltonoperators unter Eichtransformation. Dieser lautet für das Elektron im Feld des Monopols (minimale Kopplung):
1 e ~ 2
H=
p~ − A
.
(52)
2m
c
Eichtransformationen ändern die Physik nicht, d.h. zwischen den Zuständen |ψ1 i
und |ψ2 i, welche die Schrödingergleichung mit den Hamiltonoperatoren H1 bzw.
H2 und den Potentialen A~1 bzw. A~2 lösen, muss es eine unitäre Transformation
U geben, sodass
|ψ2 i = U |ψ1 i.
(53)
Die beobachtbaren Größen, in diesem Fall die Energie, dürfen sich nicht ändern,
was folgende Konsequenz für die Transformationseigenschaften des Hamiltonoperators hat:
hH1 i = hH2 i
⇔
hψ1 |H1 |ψ1 i = hψ2 |H2 |ψ2 i
†
= hψ1 |U H2 U |ψ1 i
⇒
H1
7
†
= U H2 U
(54)
(55)
(56)
U † H2 U
=
!
=
i2
e
1 †
e 2
e
1 h †
U p~ − A~1 − ∇χ U
U p~ − A~2 U =
2m
c
2m
c
c
e ~ 2
1 p~ − A1 = H1
(57)
2m
c
Zwei Zustände sind physikalisch äquivalent, wenn sie sich nur um eine komplexe
Phase unterscheiden, daher wählen wir als Ansatz U = exp (−iΛ(x)), wobei Λ
irgendeine skalare Funktion von x ist:
e
e~
e~ e
e
!
− ∇χ = p~ − A
(58)
eiΛ p~ − A~1 − ∇χ e−iΛ = p~ − ~∇Λ − A
c
c
c
c
c
e
⇒ Λ = − χ
(59)
~c
e
⇒ U = exp i χ(x)
(60)
~c
Das Eichfeld im Überlappungsbereich
der beiden Gebiete ist χ = −2gϕ, d.h.
U = exp −i 2eg
ϕ
.
Durchläuft
man
in
diesem Überlappungsbereich n-mal eine
~c
geschlossene Kurve um die z-Achse, so ändert sich ϕ um 2πn. Wegen der Eindeutigkeit des Zustandskets im Überlappungsbereich, muss aber für diesen Fall
U = 1 sein, die Phase ist demnach ein ganzzahliges Vielfaches von 2π:
2eg
2eg
2πn
= 1 ⇒
∈Z
(61)
exp −i
~c
~c
Es folgt also, dass das Produkt von elektrischer Ladung und magnetischer
Polstärke quantisiert ist, d.h. die Existenz eines kleinsten magnetischen Monopols im Universum impliziert die Existenz einer elektrischen Elementarladung
und umgekehrt. Dies ist die weitaus attraktivste Konsequenz der Annahme magnetischer Monopole, da sie eine einfache Erklärung für die Existenz einer kleinsten elektrischen Ladung liefern würde.
eg =
4
~c
k
2
für k ∈ Z
(62)
Fazit und experimenteller Befund
Die in diesem Vortrag präsentierten theoretischen Argumente aus der Maxwelltheorie und der (nichtrelativistischen) Quantenmechanik lassen die Existenz magnetischer Monopole ausdrücklich zu. Aus Gründen der Ästhetik erwartet man,
dass solche Pole existieren. Sie sind teilweise sogar fester Bestandteil einiger
GUTs, in denen diese zwangsläufig als topologische Defekte in der Raumzeit
auftreten. Die von diesen Theorien vorhergesagte Masse des Monopols liegt im
2
Bereich von 1016 GeV/c , was der Masse eines Bakteriums entspricht. Heutige
Teilchenbeschleuniger (LHC) erreichen allerdings nur Energien von ca. 103 GeV.
Dermaßen hohe Energien wurden jedoch am Anfang unseres Universums erreicht, insofern könnten sich heute noch in dieser frühen Phase erzeugte kosmische Monopole durchs All bewegen und die Erde passieren. Man schätzt, dass
ungefähr alle 31700 Jahre auf einen Quadratmeter Erdoberfläche ein Monopol
trifft. Cabrera detektierte am 14. Februar 1982 (daher manchmal auch scherzhaft als Valentinsmonopol“ bezeichnet) mittels eines SQUID3 das Auftreffen
”
3 superconducting quantum interference device - im Wesentlichen ein supraleitender Ring,
mit dem geringste Änderungen des Magnetfeldes gemessen werden können.
8
eines magnetischen Monopols auf die Erde. Das Resultat konnte allerdings seither nicht reproduziert werden. Dennoch hat Cabrera seine Veröffentlichung bis
heute nicht zurückgezogen. Alles in allem konnte die Existenz magnetischer Monopole bisher nicht experimentell bestätigt werden.
Literatur
[1] P.A.M. Dirac, Phys. Rev. 74 (1948) 817
[2] E. Amaldi, On the Dirac magnetic poles, in: Old and new problems in
elementary particles, Hrsg. G. Puppi, Acad. Press, NY 1968
[3] Ya. Shnir, Magnetic Monopoles, Springer Heidelberg, 2005
[4] http://theory.gsi.de/ vanhees/faq/qm.html
9
Herunterladen